2007 AMC 12A #25, Intermediate Algebra #11.58
by djmathman, Jul 22, 2012, 12:10 AM
These are two quite interesting problems.
Solution
Solution
2007 AMC 12A #25 wrote:
Call a set of integers spacy if it contains no more than one out of any three consecutive integers. How many subsets of
including the empty set, are spacy?



Solution
First note that the maximum number of elements in a spacy set can be
. Thus, we break the problem into cases:
CASE 0: The Empty Set
The problem says to include this set, so we must. There is
set in this case.
CASE 1: One Element
We can choose any of the twelve numbers in the set for a total of
sets in this case.
CASE 2: Two Elements
This is where things get tricky. According to the spacy condition, each number
in a spacy set, where
, must be placed such that
for
This is because if we pick the number
to be in the set, we can not have any of the numbers
in the set as well. How do we count the number of sets with such a restriction? The answer is to simply remove it.
Let's consider a set of
as shown:
![\[\text{XXXXXXXXXX}\]](//latex.artofproblemsolving.com/4/f/f/4ff219a2e86c9f44a778fd4598bf222cb4ae78d8.png)
I claim that the number of elements in the entirety of Case 2 is equal to the number of ways to select two
from this set of
. Why is this the case? Well, once we select the two
, we can squeeze two
in between our chosen
. An example is shown here:
![\[\text{XXXXX}\boxed{\text{X}}\boxed{\text{X}}\text{XXX}\implies \text{XXXXX}\boxed{\text{X}}\color{red}\text{XX}\color{black}\boxed{\text{X}}\text{XXX}\]](//latex.artofproblemsolving.com/1/a/3/1a3d45ec2e9dc49fbfdf5e9acd18ce8952f11c2f.png)
By adding these letters in this location, we separate our two selections by at least
, all while having a total of
elements, which is what we want! Therefore, the 1-1 correspondence is established, and the number of sets in Case 2 is simply
.
CASE 3: Three Elements
We do the same thing, except start off with
instead of
. This is because there are two places to put two
when we select three of them, and we want the total number of
to still be
at the end. Am example is shown here:
![\[\text{XX}\boxed{\text{X}}\text{X}\boxed{\text{X}}\boxed{\text{X}}\text{XX}\implies \text{XX}\boxed{\text{X}}\color{red}\text{XX}\color{black}\text{X}\boxed{\text{X}}\color{red}\text{XX}\color{black}\boxed{\text{X}}\text{XX}\]](//latex.artofproblemsolving.com/b/c/d/bcd1f806c872b2a24632f4f1c6577e497f38b0da.png)
By this 1-1 correspondence, the number of sets in Case 3 is the number of ways to select
from a set of
, which is simply
.
CASE 4: Four Elements
I think you know what the pattern is by now. We only need six
to begin with now, and we want to select four of them. The number of sets here is
.
Adding all of these values up gives us our desired answer:
.

CASE 0: The Empty Set
The problem says to include this set, so we must. There is

CASE 1: One Element
We can choose any of the twelve numbers in the set for a total of

CASE 2: Two Elements
This is where things get tricky. According to the spacy condition, each number






Let's consider a set of


![\[\text{XXXXXXXXXX}\]](http://latex.artofproblemsolving.com/4/f/f/4ff219a2e86c9f44a778fd4598bf222cb4ae78d8.png)
I claim that the number of elements in the entirety of Case 2 is equal to the number of ways to select two





![\[\text{XXXXX}\boxed{\text{X}}\boxed{\text{X}}\text{XXX}\implies \text{XXXXX}\boxed{\text{X}}\color{red}\text{XX}\color{black}\boxed{\text{X}}\text{XXX}\]](http://latex.artofproblemsolving.com/1/a/3/1a3d45ec2e9dc49fbfdf5e9acd18ce8952f11c2f.png)
By adding these letters in this location, we separate our two selections by at least




CASE 3: Three Elements
We do the same thing, except start off with





![\[\text{XX}\boxed{\text{X}}\text{X}\boxed{\text{X}}\boxed{\text{X}}\text{XX}\implies \text{XX}\boxed{\text{X}}\color{red}\text{XX}\color{black}\text{X}\boxed{\text{X}}\color{red}\text{XX}\color{black}\boxed{\text{X}}\text{XX}\]](http://latex.artofproblemsolving.com/b/c/d/bcd1f806c872b2a24632f4f1c6577e497f38b0da.png)
By this 1-1 correspondence, the number of sets in Case 3 is the number of ways to select




CASE 4: Four Elements
I think you know what the pattern is by now. We only need six


Adding all of these values up gives us our desired answer:

Intermediate Algebra #11.58 wrote:
Let
be a positive integer. Prove that
.


Solution
We proceed by induction.
BASE CASE:
Here, we have
, which is true.
INDUCTIVE STEP:
Assume that
for some
.
Remark that since
, it is obvious that
. Adding
to both sides gives us
, so
. Rearranging this gives us
![\[\dfrac{1}{(2k+1)(2k+2)}\geq \dfrac{2}{(3k+1)(3k+4)}.\]](//latex.artofproblemsolving.com/3/d/5/3d5073893fe43413e96b2902bdcd2eb2d4abcc3f.png)
Next note that the LHS can be rewritten as
.
Also note that the RHS can be rewritten as
. Hence our inequality becomes
![\[\dfrac{1}{2k+1}+\dfrac{1}{2(k+1)}-\dfrac{1}{k+1}\geq \dfrac{2(k+1)}{3(k+1)+1}-\dfrac{2k}{3k+1}.\]](//latex.artofproblemsolving.com/2/c/2/2c2f56923efe04a466c9d10dfa67158f4636d5bd.png)
Adding this to our induction step gives us
![\[\left(\dfrac{1}{k+1}+\dfrac{1}{k+2}+\cdots+\dfrac{1}{2k}\right)+\left(\dfrac{1}{2k+1}+\dfrac{1}{2(k+1)}-\dfrac{1}{k+1}\right)\geq \dfrac{2k}{3k+1}+\left(\dfrac{2k+2}{3k+4}-\dfrac{2k}{3k+1}\right),\]](//latex.artofproblemsolving.com/f/1/1/f1187bebc4e308df98cdd0b05bef8ee979960bfd.png)
and simplifying this gives us
![\[\dfrac{1}{k+2}+\dfrac{1}{k+3}+\cdots+\dfrac{1}{2k+1}+\dfrac{1}{2(k+1)}\geq \dfrac{2(k+1)}{3(k+1)+1}.\]](//latex.artofproblemsolving.com/a/3/7/a370d27526dbb027f1a48b2e384b534d1b3dc106.png)
We assumed the hypothesis was true for
, and we have proven it true for
. Our induction is complete. 
BASE CASE:

Here, we have

INDUCTIVE STEP:
Assume that


Remark that since





![\[\dfrac{1}{(2k+1)(2k+2)}\geq \dfrac{2}{(3k+1)(3k+4)}.\]](http://latex.artofproblemsolving.com/3/d/5/3d5073893fe43413e96b2902bdcd2eb2d4abcc3f.png)
Next note that the LHS can be rewritten as

Also note that the RHS can be rewritten as

![\[\dfrac{1}{2k+1}+\dfrac{1}{2(k+1)}-\dfrac{1}{k+1}\geq \dfrac{2(k+1)}{3(k+1)+1}-\dfrac{2k}{3k+1}.\]](http://latex.artofproblemsolving.com/2/c/2/2c2f56923efe04a466c9d10dfa67158f4636d5bd.png)
Adding this to our induction step gives us
![\[\left(\dfrac{1}{k+1}+\dfrac{1}{k+2}+\cdots+\dfrac{1}{2k}\right)+\left(\dfrac{1}{2k+1}+\dfrac{1}{2(k+1)}-\dfrac{1}{k+1}\right)\geq \dfrac{2k}{3k+1}+\left(\dfrac{2k+2}{3k+4}-\dfrac{2k}{3k+1}\right),\]](http://latex.artofproblemsolving.com/f/1/1/f1187bebc4e308df98cdd0b05bef8ee979960bfd.png)
and simplifying this gives us
![\[\dfrac{1}{k+2}+\dfrac{1}{k+3}+\cdots+\dfrac{1}{2k+1}+\dfrac{1}{2(k+1)}\geq \dfrac{2(k+1)}{3(k+1)+1}.\]](http://latex.artofproblemsolving.com/a/3/7/a370d27526dbb027f1a48b2e384b534d1b3dc106.png)
We assumed the hypothesis was true for


